Difference between revisions of "2021 AMC 12B Problems/Problem 20"

m (Solution 4 (Division Analysis, Similar Approach but Different Explanation from Solution 1))
m (Solution 4 (Complex Numbers))
(36 intermediate revisions by 2 users not shown)
Line 2: Line 2:
 
Let <math>Q(z)</math> and <math>R(z)</math> be the unique polynomials such that<cmath>z^{2021}+1=(z^2+z+1)Q(z)+R(z)</cmath>and the degree of <math>R</math> is less than <math>2.</math> What is <math>R(z)?</math>
 
Let <math>Q(z)</math> and <math>R(z)</math> be the unique polynomials such that<cmath>z^{2021}+1=(z^2+z+1)Q(z)+R(z)</cmath>and the degree of <math>R</math> is less than <math>2.</math> What is <math>R(z)?</math>
  
<math>\textbf{(A) }-z \qquad \textbf{(B) }-1 \qquad \textbf{(C) }2021\qquad \textbf{(D) }z+1 \qquad \textbf{(E) }2z+1</math>
+
<math>\textbf{(A) }{-}z \qquad \textbf{(B) }{-}1 \qquad \textbf{(C) }2021\qquad \textbf{(D) }z+1 \qquad \textbf{(E) }2z+1</math>
  
==Solution 1==
+
==Solution 1 (Difference of Cubes)==
===Solution 1.1===
+
Let <math>z=s</math> be a root of <math>z^2+z+1</math> so that <math>s^2+s+1=0.</math> It follows that <cmath>(s-1)\left(s^2+s+1\right)=s^3-1=0,</cmath> from which <math>s^3=1,</math> but <math>s\neq1.</math>
 +
 
 +
Note that
 +
<cmath>\begin{align*}
 +
s^{2021}+1 &= s^{3\cdot673+2}+1 \\
 +
&= (s^3)^{673}\cdot s^2+1 \\
 +
&= s^2+1 \\
 +
&= \left(s^2+s+1\right)-s \\
 +
&= -s.
 +
\end{align*}</cmath>
 +
Since <math>z^{2021}+1=-z</math> for each root <math>z=s</math> of <math>z^2+z+1,</math> the remainder when <math>z^{2021}+1</math> is divided by <math>z^2+z+1</math> is <math>R(z)=\boxed{\textbf{(A) }{-}z}.</math>
 +
 
 +
~MRENTHUSIASM
 +
 
 +
==Solution 2 (Finds Q(z) Using Patterns)==
 +
 
 +
Note that the equation above is in the form of polynomial division, with <math>z^{2021}+1</math> being the dividend, <math>z^2+z+1</math> being the divisor, and <math>Q(x)</math> and <math>R(x)</math> being the quotient and remainder respectively. Since the degree of the dividend is <math>2021</math> and the degree of the divisor is <math>2</math>, that means the degree of the quotient is <math>2021-2 = 2019</math>. Note that <math>R(x)</math> can't influence the degree of the right hand side of this equation since its degree is either <math>1</math> or <math>0</math>. Since the coefficients of the leading term in the dividend and the divisor are both <math>1</math>, that means the coefficient of the leading term of the quotient is also <math>1</math>. Thus, the leading term of the quotient is <math>z^{2019}</math>. Multiplying <math>z^{2019}</math> by the divisor gives <math>z^{2021}+z^{2020}+z^{2019}</math>. We have our <math>z^{2021}</math> term but we have these unnecessary terms like <math>z^{2020}</math>. We can get rid of these terms by adding <math>-z^{2018}</math> to the quotient to cancel out these terms, but this then gives us <math>z^{2021}-z^{2018}</math>. Our first instinct will probably be to add <math>z^{2017}</math>, but we can't do this as although this will eliminate the <math>-z^{2018}</math> term, it will produce a <math>z^{2019}</math> term. Since no other term of the form <math>z^n</math> where <math>n</math> is an integer less than <math>2017</math> will produce a <math>z^{2019}</math> term when multiplied by the divisor, we can't add <math>z^{2017}</math> to the quotient. Instead, we can add <math>z^{2016}</math> to the coefficient to get rid of the <math>-z^{2018}</math> term. Continuing this pattern, we get the quotient as <cmath>z^{2019}-z^{2018}+z^{2016}-z^{2015}+\cdots-z^2+1.</cmath>
 +
The last term when multiplied with the divisor gives <math>z^2+z+1</math>. This will get rid of the <math>-z^2</math> term but will produce the expression <math>z+1</math>, giving us the dividend as <math>z^{2021}+z+1</math>. Note that the dividend we want is of the form <math>z^{2021}+1</math>. Therefore, our remainder will have to be <math>-z</math> in order to get rid of the <math>z</math> term in the expression and give us <math>z^{2021}+1</math>, which is what we want. Therefore, the remainder is <math>\boxed{\textbf{(A) }{-}z}.</math>
 +
 
 +
~ rohan.sp ~rocketsri
 +
 
 +
==Solution 3 (Modular Arithmetic in Polynomials)==
 
Note that
 
Note that
 
<cmath>z^3-1\equiv 0\pmod{z^2+z+1}</cmath>
 
<cmath>z^3-1\equiv 0\pmod{z^2+z+1}</cmath>
Line 14: Line 35:
 
So <math>F(z) = z^2+1</math>, and
 
So <math>F(z) = z^2+1</math>, and
 
<cmath>R(z)\equiv F(z) \equiv -z\pmod{z^2+z+1}</cmath>
 
<cmath>R(z)\equiv F(z) \equiv -z\pmod{z^2+z+1}</cmath>
The answer is <math>\boxed{\textbf{(A) }-z}.</math>
+
The answer is <math>\boxed{\textbf{(A) }{-}z}.</math>
  
===Solution 1.2 (More Thorough Version of Solution 1.1)===
+
==Solution 4 (Complex Numbers)==
Instead of dealing with a nasty <math>z^2+z+1</math>, we can instead deal with the nice <math>z^3 - 1</math>, as <math>z^2+z+1</math> is a factor of <math>z^3-1</math>. Then, we try to see what <math>\frac{z^{2021} + 1}{z^3 - 1}</math> is. Of course, we will need a <math>z^{2018}</math>, getting <math>z^{2021} - z^{2018}</math>. Then, we've gotta get rid of the <math>z^{2018}</math> term, so we add a <math>z^{2015}</math>, to get <math>z^{2021} - z^{2015}</math>. This pattern continues, until we add a <math>z^2</math> to get rid of <math>z^5</math>, and end up with <math>z^{2021} - z^2</math>. We can't add anything more to get rid of the <math>z^2</math>, so our factor is <math>z^{2018} + z^{2015} + z^{2012} + \cdots + z^2</math>. Then, to get rid of the <math>z^2</math>, we must have a remainder of <math>+z^2</math>, and to get the <math>+1</math> we have to also have a <math>+1</math> in the remainder. So, our product is <cmath>z^{2021}+1= (z^3-1)(z^{2018} + z^{2015} + \cdots + z^2) + z^2+1.</cmath> Then, our remainder is <math>z^2+1</math>. The remainder when dividing by <math>z^3-1</math> must be the same when dividing by <math>z^2+z+1</math>, modulo <math>z^2+z+1</math>. So, we have that <math>z^2 + 1 \equiv R(z) \pmod{z^2+z+1}</math>, or <math>R(z) \equiv -z\pmod{z^2+z+1}</math>. This corresponds to answer choice <math>\boxed{\textbf{(A)} ~ -z}</math>. ~rocketsri
+
One thing to note is that <math>R(z)</math> takes the form of <math>Az + B</math> for some constants <math>A</math> and <math>B.</math>
 
+
Note that the roots of <math>z^2 + z + 1</math> are part of the solutions of <math>z^3 -1 = 0.</math>
==Solution 2 (Complex numbers)==
 
One thing to note is that <math>R(z)</math> takes the form of <math>Az + B</math> for some constants A and B.
 
Note that the roots of <math>z^2 + z + 1</math> are part of the solutions of <math>z^3 -1 = 0</math>
 
 
They can be easily solved with roots of unity:
 
They can be easily solved with roots of unity:
<cmath>z^3 = 1</cmath>
 
<cmath>z^3 = e^{i 0}</cmath>
 
<cmath>z = e^{i 0}, e^{i \frac{2\pi}{3}}, e^{i -\frac{2\pi}{3}}</cmath> <cmath>\newline</cmath>
 
Obviously the right two solutions are the roots of <math>z^2 + z + 1 = 0</math>
 
We substitute <math>e^{i \frac{2\pi}{3}}</math> into the original equation, and <math>z^2 + z + 1</math> becomes 0. Using De Moivre's theorem, we get:
 
<cmath>e^{i\frac{4042\pi}{3}} + 1 = A \cdot e^{i \frac{2\pi}{3}} + B</cmath>
 
<cmath>e^{i\frac{4\pi}{3}} + 1 = A \cdot e^{i \frac{2\pi}{3}} + B</cmath>
 
Expanding into rectangular complex number form:
 
<cmath>\frac{1}{2} - \frac{\sqrt{3}}{2} i = (-\frac{1}{2}A + B) + \frac{\sqrt{3}}{2} i A</cmath>
 
Comparing the real and imaginary parts, we get:
 
<cmath>A = -1, B = 0</cmath>
 
The answer is <math>\boxed{\textbf{(A) }-z}</math>. ~Jamess2022(burntTacos;-;)
 
 
==Solution 3 (similar to Solution 1.2)==
 
 
Note that the equation above is in the form of polynomial division, with <math>z^{2021}+1</math> being the dividend, <math>z^2+z+1</math> being the divisor, and <math>Q(x)</math> and <math>R(x)</math> being the quotient and remainder respectively. Since the degree of the dividend is <math>2021</math> and the degree of the divisor is <math>2</math>, that means the degree of the quotient is <math>2021-2 = 2019</math>. Note that R(x) can't influence the degree of the right hand side of this equation since its degree is either <math>1</math> or <math>0</math>. Since the coefficients of the leading term in the dividend and the divisor are both <math>1</math>, that means the coefficient of the leading term of the quotient is also <math>1</math>. Thus, the leading term of the quotient is <math>z^{2019}</math>. Multiplying <math>z^{2019}</math> by the divisor gives <math>z^{2021}+z^{2020}+z^{2019}</math>. We have our <math>z^{2021}</math> term but we have these unnecessary terms like <math>z^{2020}</math>. We can get rid of these terms by adding <math>-z^{2018}</math> to the quotient to cancel out these terms, but this then gives us <math>z^{2021}-z^{2018}</math>. Our first instinct will probably be to add <math>z^{2017}</math>, but we can't do this as although this will eliminate the <math>-z^{2018}</math> term, it will produce a <math>z^{2019}</math> term. Since no other term of the form <math>z^n</math> where <math>n</math> is an integer less than <math>2017</math> will produce a <math>z^{2019}</math> term when multiplied by the divisor, we can't add <math>z^{2017}</math> to the quotient. Instead, we can add <math>z^{2016}</math> to the coefficient to get rid of the <math>-z^{2018}</math> term. Continuing this pattern, we get the quotient as <cmath>z^{2019}-z^{2018}+z^{2016}-z^{2015}+....-z^2+1.</cmath>
 
The last term when multiplied with the divisor gives <math>z^2+z+1</math>. This will get rid of the <math>-z^2</math> term but will produce the expression <math>z+1</math>, giving us the dividend as <math>z^{2021}+z+1</math>. Note that the dividend we want is of the form <math>z^{2021}+1</math>. Therefore, our remainder will have to be <math>-z</math> in order to get rid of the <math>z</math> term in the expression and give us <math>z^{2021}+1</math>, which is what we want. Therefore, the remainder is <math>\boxed{\textbf{(A) }-z \qquad}</math>
 
 
~ rohan.sp
 
 
==Solution 4 (Division Analysis, Similar Approach but Different Explanation from Solution 1)==
 
By the difference of cubes or the short geometric series, we get <cmath>\left(z-1\right)\left(z^2+z+1\right)=z^3-1.</cmath>
 
We rewrite <math>\left(z^{2021}+1\right)\div\left(z^3-1\right)</math> by the polynomial division algorithm:
 
<cmath>z^{2021}+1=\left(z^3-1\right)Q'(z)+R'(z),  \ \ \ \ \ \ \ \ \ \ \ \ (*)</cmath>
 
where <math>Q'(z)</math> and <math>R'(z)</math> are unique polynomials such that <math>\text{deg}\left(R'(z)\right)<3.</math> Taking <math>(*)</math> in modulo <math>z^3-1</math> (in which <math>z^3=1</math>), we have <cmath>z^{2021}+1=z^{3\cdot673+2}+1=z^{3\cdot673}z^2+1=\left(z^3\right)^{673}z^2+1=z^2+1=R'(z).</cmath>
 
Substituting <math>R'(z)=z^2+1</math> back into <math>(*)</math> gives
 
 
<cmath>\begin{align*}
 
<cmath>\begin{align*}
z^{2021}+1&=\left(z^3-1\right)Q'(z)+\left(z^2+1\right) \\
+
z^3 &= 1 \\
&=(z-1)\left(z^2+z+1\right)Q'(z)+\left(z^2+1\right) \\
+
z^3 &= e^{i 0} \\
&=\left(z^2+z+1\right)\left[(z-1)Q'(z)\right]+\left(z^2+1\right), \ \ \ \ \ \ \ \ \ \ \ \ (**)
+
z &= e^{i 0}, e^{i \frac{2\pi}{3}}, e^{i\left(-\frac{2\pi}{3}\right)}.
 
\end{align*}</cmath>
 
\end{align*}</cmath>
which almost resembles to the original equation <cmath>z^{2021}+1=(z^2+z+1)Q(z)+R(z).</cmath> Since we require that <math>\text{deg}\left(R(z)\right)<2,</math> the divisor <math>z^2+z+1</math> goes into the remaining <math>z^2+1</math> for one more time. Rewriting <math>(**)</math> produces
+
Obviously the right two solutions are the roots of <math>z^2 + z + 1 = 0.</math>
 +
We substitute <math>e^{i \frac{2\pi}{3}}</math> into the original equation, and <math>z^2 + z + 1</math> becomes <math>0.</math> Using De Moivre's Theorem, we get
 
<cmath>\begin{align*}
 
<cmath>\begin{align*}
z^{2021}+1&=\left(z^2+z+1\right)\left[(z-1)Q'(z)+1\right]+\left(\left(z^2+1\right)-\left(z^2+z+1\right)\right) \\
+
e^{i\frac{4042\pi}{3}} + 1 &= A \cdot e^{i \frac{2\pi}{3}} + B \\
&=\left(z^2+z+1\right)\left[(z-1)Q'(z)+1\right]+(-z),
+
e^{i\frac{4\pi}{3}} + 1 &= A \cdot e^{i \frac{2\pi}{3}} + B.
 
\end{align*}</cmath>
 
\end{align*}</cmath>
from which <math>R(z)=\boxed{\textbf{(A) }-z}.</math>
+
Expanding into rectangular complex number form:
 +
<cmath>\frac{1}{2} - \frac{\sqrt{3}}{2} i = \left(-\frac{1}{2}A + B\right) + \frac{\sqrt{3}}{2} A i.</cmath>
 +
Comparing the real and imaginary parts, we get <math>(A,B) = (-1,0).</math> So, the answer is <math>\boxed{\textbf{(A) }{-}z}.</math>
  
~MRENTHUSIASM
+
~Jamess2022 (burntTacos ;-))
  
== Video Solution by OmegaLearn (Using Modular Arithmetic and Meta-solving) ==
+
== Video Solution by OmegaLearn (Using Modular Arithmetic and Meta-Solving) ==
 
https://youtu.be/nnjr17q7fS0
 
https://youtu.be/nnjr17q7fS0
  
==Video Solution using long division(not brutal)==
+
==Video Solution (Long Division, Not Brutal)==
 
https://youtu.be/kxPDeQRGLEg
 
https://youtu.be/kxPDeQRGLEg
 
~hippopotamus1
 
~hippopotamus1

Revision as of 01:31, 24 November 2021

Problem

Let $Q(z)$ and $R(z)$ be the unique polynomials such that\[z^{2021}+1=(z^2+z+1)Q(z)+R(z)\]and the degree of $R$ is less than $2.$ What is $R(z)?$

$\textbf{(A) }{-}z \qquad \textbf{(B) }{-}1 \qquad \textbf{(C) }2021\qquad \textbf{(D) }z+1 \qquad \textbf{(E) }2z+1$

Solution 1 (Difference of Cubes)

Let $z=s$ be a root of $z^2+z+1$ so that $s^2+s+1=0.$ It follows that \[(s-1)\left(s^2+s+1\right)=s^3-1=0,\] from which $s^3=1,$ but $s\neq1.$

Note that \begin{align*} s^{2021}+1 &= s^{3\cdot673+2}+1 \\ &= (s^3)^{673}\cdot s^2+1 \\ &= s^2+1 \\ &= \left(s^2+s+1\right)-s \\ &= -s. \end{align*} Since $z^{2021}+1=-z$ for each root $z=s$ of $z^2+z+1,$ the remainder when $z^{2021}+1$ is divided by $z^2+z+1$ is $R(z)=\boxed{\textbf{(A) }{-}z}.$

~MRENTHUSIASM

Solution 2 (Finds Q(z) Using Patterns)

Note that the equation above is in the form of polynomial division, with $z^{2021}+1$ being the dividend, $z^2+z+1$ being the divisor, and $Q(x)$ and $R(x)$ being the quotient and remainder respectively. Since the degree of the dividend is $2021$ and the degree of the divisor is $2$, that means the degree of the quotient is $2021-2 = 2019$. Note that $R(x)$ can't influence the degree of the right hand side of this equation since its degree is either $1$ or $0$. Since the coefficients of the leading term in the dividend and the divisor are both $1$, that means the coefficient of the leading term of the quotient is also $1$. Thus, the leading term of the quotient is $z^{2019}$. Multiplying $z^{2019}$ by the divisor gives $z^{2021}+z^{2020}+z^{2019}$. We have our $z^{2021}$ term but we have these unnecessary terms like $z^{2020}$. We can get rid of these terms by adding $-z^{2018}$ to the quotient to cancel out these terms, but this then gives us $z^{2021}-z^{2018}$. Our first instinct will probably be to add $z^{2017}$, but we can't do this as although this will eliminate the $-z^{2018}$ term, it will produce a $z^{2019}$ term. Since no other term of the form $z^n$ where $n$ is an integer less than $2017$ will produce a $z^{2019}$ term when multiplied by the divisor, we can't add $z^{2017}$ to the quotient. Instead, we can add $z^{2016}$ to the coefficient to get rid of the $-z^{2018}$ term. Continuing this pattern, we get the quotient as \[z^{2019}-z^{2018}+z^{2016}-z^{2015}+\cdots-z^2+1.\] The last term when multiplied with the divisor gives $z^2+z+1$. This will get rid of the $-z^2$ term but will produce the expression $z+1$, giving us the dividend as $z^{2021}+z+1$. Note that the dividend we want is of the form $z^{2021}+1$. Therefore, our remainder will have to be $-z$ in order to get rid of the $z$ term in the expression and give us $z^{2021}+1$, which is what we want. Therefore, the remainder is $\boxed{\textbf{(A) }{-}z}.$

~ rohan.sp ~rocketsri

Solution 3 (Modular Arithmetic in Polynomials)

Note that \[z^3-1\equiv 0\pmod{z^2+z+1}\] so if $F(z)$ is the remainder when dividing by $z^3-1$, \[F(z)\equiv R(z)\pmod{z^2+z+1}.\] Now, \[z^{2021}+1= (z^3-1)(z^{2018} + z^{2015} + \cdots + z^2) + z^2+1\] So $F(z) = z^2+1$, and \[R(z)\equiv F(z) \equiv -z\pmod{z^2+z+1}\] The answer is $\boxed{\textbf{(A) }{-}z}.$

Solution 4 (Complex Numbers)

One thing to note is that $R(z)$ takes the form of $Az + B$ for some constants $A$ and $B.$ Note that the roots of $z^2 + z + 1$ are part of the solutions of $z^3 -1 = 0.$ They can be easily solved with roots of unity: \begin{align*} z^3 &= 1 \\ z^3 &= e^{i 0} \\ z &= e^{i 0}, e^{i \frac{2\pi}{3}}, e^{i\left(-\frac{2\pi}{3}\right)}. \end{align*} Obviously the right two solutions are the roots of $z^2 + z + 1 = 0.$ We substitute $e^{i \frac{2\pi}{3}}$ into the original equation, and $z^2 + z + 1$ becomes $0.$ Using De Moivre's Theorem, we get \begin{align*} e^{i\frac{4042\pi}{3}} + 1 &= A \cdot e^{i \frac{2\pi}{3}} + B \\ e^{i\frac{4\pi}{3}} + 1 &= A \cdot e^{i \frac{2\pi}{3}} + B. \end{align*} Expanding into rectangular complex number form: \[\frac{1}{2} - \frac{\sqrt{3}}{2} i = \left(-\frac{1}{2}A + B\right) + \frac{\sqrt{3}}{2} A i.\] Comparing the real and imaginary parts, we get $(A,B) = (-1,0).$ So, the answer is $\boxed{\textbf{(A) }{-}z}.$

~Jamess2022 (burntTacos ;-))

Video Solution by OmegaLearn (Using Modular Arithmetic and Meta-Solving)

https://youtu.be/nnjr17q7fS0

Video Solution (Long Division, Not Brutal)

https://youtu.be/kxPDeQRGLEg ~hippopotamus1

~ pi_is_3.14

See Also

2021 AMC 12B (ProblemsAnswer KeyResources)
Preceded by
Problem 19
Followed by
Problem 21
1 2 3 4 5 6 7 8 9 10 11 12 13 14 15 16 17 18 19 20 21 22 23 24 25
All AMC 12 Problems and Solutions

The problems on this page are copyrighted by the Mathematical Association of America's American Mathematics Competitions. AMC logo.png